You are on page 1of 64

MBE CONTRACTS

Learning Objectives

After this lecture, you will be able to:

1. Identify the sources of contract law.

2. Define the essential elements of a legally enforceable contract.

3. Explain common substitutes and alternatives to consideration and legally valid contracts.

4. List defenses to contract formation.

5. Analyze and evaluate performance issues.

6. Explain the meaning and effects of breach of contract.

7. Apply rules pertaining to legal damages and other breach of contract remedies.

8. Evaluate third-party issues.

NOTE: This lecture covers heavily tested topics on the bar exam and is not intended to be your sole
source of information on the subject matter. Please refer to your Subjects Outline Materials and Bar
Points to gain a greater understanding of the nuances within the subject.

MBE is a trademark of the National Conference of Bar Examiners.

1
MBE CONTRACTS
SOURCES OF CONTRACT LAW
1) Governing Law  Common Law or UCC?
 UCC governs “sales of goods”. If UCC applies, it trumps any contrary common law rule
o Sales = any transaction in which the seller transfers title of goods to buyer
o Goods = any movable item. Does not include intangibles (goodwill, IP), money, legal claims,
services, or real property
 Hybrid cases (involves both sale of goods + service contracts (Ks)) - determined by predominant
purpose of the transaction (Majority rule)
o Contract language – Was K described as a goods K?
o Nature of supplier’s business – i.e., Best Buy sells goods
o Value of goods v. services – i.e., $30 in goods but $2 services?
 Even if UCC, common law still applies unless UCC contradicts it
 General categories of contractual obligations:
o Express = oral and written expressions of the agreement
o Implied-in-fact = formed by conduct rather than words
EXAMPLE: Homeowner hires a plumber to fix a leak, but because of the urgency of the service, the
parties do not discuss the price of the work performed. Upon completion of the work, Homeowner
has an implied-in-fact obligation to pay the plumber the reasonable value of the services rendered.
o Implied-in-law = arises where one party bestows benefit on another and it would be unjust not to
pay the reasonable value of the benefit
EXAMPLE: A surgeon who performs emergency surgery on an unconscious patient creates an
implied-in-law obligation to the patient.
EXAMPLE: A merchant who mistakenly delivers goods to the wrong party may create an implied-
in-law obligation on the wrong party.

Notes:

2
MBE CONTRACTS
FORMATION
1) Offer: to be an offer, party’s communication must meet two elements:
 Outward manifestation (oral, written, or via conduct)
EXAMPLE: A seemingly serious offer to sell real property made in secret jest is nonetheless an offer.
EXAMPLE: A proposal to sell at a price that a reasonable person would regard as “much too good to
be true” (e.g., “new HDTVs for $8.99”) DOES NOT constitute an offer.
 Signal that acceptance will conclude the deal (power of acceptance)
EXAMPLE: “I will sell you my car if you’ll pay me $5,000 cash.” This is an offer because it expresses a
willingness to conclude the deal if the other party pays the required $5,000.
EXAMPLE: “Yes, I’d be willing to sell you my car, but what are you willing to pay for it?” This is NOT
AN OFFER because the communicating party is obviously reserving the right to decide whether she
likes the price suggested by the other party.

2) Multiple Offers
 Commercial Advertisements
o American Rule: Ads, catalogs, price lists are invitations for offers, since responses may exceed
available supply of goods or services
 Exception: Language that identifies who gets limited supply of goods even if there is an excess
demand (i.e. first come, first served, or first 10 customers)
 Reward Offers = Offers because they are communications that promise $ in exchange for performance
of specific tasks
EXAMPLE: $500 for finding and returning my lost dog.

Notes:

3
MBE CONTRACTS
3) Termination – Offer creates the power of acceptance in the offeree, 4 ways to terminate this power:
 Lapse: after time stated in offer or after a reasonable time. Reasonable time determined by: (1) subject
matter/market conditions, and (2) degree of urgency and means of transmission
o Face-to-Face Conversation Rule: an offer made in a face-to-face conversation generally lapses at
the end of the conversation
 Death or incapacity of either party after offer is made terminates the power of acceptance
 Revocation of offer by Offeror
o Offeror may revoke an offer at any time, for any reason:
 Must be revoked before acceptance + revocation must be communicated to the offeree
 Direct revocation – offeror directly communicates to offeree an intent to withdraw the offer
 Indirect revocation – 2 requirements:
o Offeror takes action that is inconsistent with the intent to go through with the offer and
o Offeree learns of such action from reliable source
EXAMPLE: While deciding whether to accept an offer to sell you a car, you learn from a
friend that the car was sold to someone else.

HYPOTHETICAL: A offers to sell real property to B. While B is considering the offer, A sells the property to C.
Oblivious to the third-party sale, B sees A on the street and yells, “I accept your offer!” Is A now contractually
bound to sell the property to B?

o Preventing Revocation
 American Rule: the offeror can revoke even if he gave a specific time to accept
 Option Contract (common law) – Elements
 Offer + Separate promise to keep it open + Valid mechanism for enforcing subsidiary
promise (consideration is most common way)
EXAMPLE: “I hereby offer to sell you Blackacre for $10,000, and in consideration for the $100
received, I hereby grant you a 30-day option on the deal.”
 Reliance/Construction: Courts will hold offers open when the offeree has detrimentally relied on
them, such as when general contractors rely on subcontractor’s bids in forming their own bids on a
project.

Notes:

4
MBE CONTRACTS

 Firm Offer under UCC – Irrevocable offer by merchant to buy or sell goods without
consideration (3 requirements):
 Offer made by a merchant (in the business of buying or selling goods) +
 In a writing signed by the merchant +
 Expressly stating it will be held open
o Irrevocable for time stated or reasonable time, BUT no longer than 3 months even if
stated otherwise

 Termination by Offeree’s Rejection (3 ways)


o Outright rejection
o Rejection via counteroffer: counteroffer = rejection + new offer (e.g., I am not willing to pay $10K for
the car, but I would happily buy your car for $9K.)
 Exception: offeree can test the waters by making a mere inquiry (e.g., $10k isn’t out of the
question but it’s a little high given the age, would you consider a lower offer?)
o Rejection via non-conforming acceptance
 Mirror image rule (common law): Acceptance must mirror the terms, and any variation is
counteroffer (and rejection of initial offer)
EXAMPLE: X offers to buy goods from Y, Y says OK, and says he expects payment in 30 days.
This will be a violation of the mirror image rule, and thus a counteroffer.
4) Offer and Acceptance under Unilateral Contracts
 Bilateral Contract = Offer seeking acceptance by a promise. A promise is being exchanged for
a promise. Once promises are exchanged, both parties are bound
 Unilateral Contract = Offer seeking performance in return (i.e., reward offers). Offeror not bound until
offeree completes performance, and offeree is NEVER bound.

HYPOTHETICAL: A says to his old friend B, “I’ll sell you my car for $10,000, but due to your credit problem,
you are going to have to pay me the money or I will sell it to someone else.” Will a promissory acceptance from
B form a binding contract with A?

o Revocation of Offer - Unilateral Contract: once offeree begins performance, an option K is created
and offeror may not revoke. Mere preparations do not create an option K, only beginning
performance.
EXAMPLE: X offers Y $100 to “ride my horse Bronco for a minute,” and while Y is putting on his
boots X revokes.
Notes:

5
MBE CONTRACTS

5) Acceptance (Common Law)


 Acceptance must mirror terms of offer + be communicated to the offeror
o 2 Exceptions to requirement that acceptance be communicated:
 Unilateral K: Acceptance is effective only by completing performance, no communication is
required unless offer provides otherwise

HYPOTHETICAL: Professor A makes the following offer to her students: “For anyone who completes this
series of exercises by Friday, I’ll provide you with a free beverage from Starbucks.” Student B completes the
exercises by Friday but does not notify the Professor until Monday. Does B’s completion of the exercises by
Friday constitute acceptance of A’s offer? What if, on the same facts, the professor had stipulated in her offer
that the students had to not only complete the series by Friday but also had to notify her that they had done so
by Friday?

 Acceptance by mail
 Common law Mailbox Rule: acceptance by mail is effective upon dispatch if properly
posted.
o Applies only to acceptances and not to any other communication (i.e., not to revocations
or rejections)
o Default rule – it applies unless the offer provides otherwise

HYPOTHETICAL: A’s offer to B stipulates that A “must hear from B by the close of business Thursday” in
order for B’s acceptance to be effective. B dispatches acceptance of the offer on Wednesday, but it doesn’t
reach A until Friday. Is B’s acceptance effective?

 The Mailbox Rule and Option Contracts


o Restatement/majority rule: mailbox rule is not applicable and acceptance is only effective
upon receipt
 Hard Case: What happens when an offeree dispatches two responses to an offer, the first
purporting to reject the offer and the second purporting to accept it?
o Mailbox rule does not govern if rejection is mailed before acceptance, and whichever
arrives first will be effective.

Notes:

6
MBE CONTRACTS
6) UCC - Acceptance by Seller’s Shipment
 Acceptance by Seller’s Shipment of Nonconforming Goods
o Seller can accept Buyer’s offer to purchase goods for prompt or current shipment in 3 ways:
 Promise to ship goods;
 Shipping conforming goods;
 Shipping non-conforming goods, unless the seller sends the shipment as an accommodation
(i.e., counteroffer)
EXAMPLE: Buyer orders 1,000 widgets from seller for immediate delivery. Seller responds by
shipping 800 widgets with an accompanying notice to the buyer explaining that the seller did not
have adequate inventory to ship 1,000 widgets and was thus shipping 800 widgets as an
accommodation to the buyer in light of the buyer’s urgent need.
7) UCC - Battle of the Forms
 UCC Rejection of the Mirror Image Rule
o An offeree’s nonconforming acceptance or confirmation (with additional terms) will operate as an
effective acceptance of the offer, thus forming a contract (not a counteroffer).
EXAMPLE: Buyer sends Seller a purchase order for 1,000 widgets at the advertised price of $10
each. Seller sends Buyer an Acknowledgment of Order form that promises delivery of the widgets
at the stated price, but also contains boilerplate language that negates warranties and limits
remedies in the event of breach. Seller’s form will operate as acceptance of Buyer’s offer and
create a binding contract despite the presence of terms that vary from Buyer’s purchase order.

Notes:

7
MBE CONTRACTS
 Effect of Additional or Different Terms
o Between merchants, the “additional” terms in offeree’s acceptance or confirmation become part of
the K EXCEPT in three circumstances:
 The offer expressly limits acceptance to its own terms
EXAMPLE: “This order expressly limits acceptance to the terms stated herein.”
 If offeror objects to the additional terms within a reasonable time
EXAMPLE: This could be accomplished through language to the effect of: “We do not accept
the binding arbitration provision set forth in your Acknowledgment of Order.”
 If the additional terms would materially alter the K
 “Material alteration” = terms that would result in “surprise or hardship if incorporated w/o
the express awareness of the other party”.
 Examples of clauses that would materially alter the contract include:
o warranty disclaimers
o clauses that materially shorten the deadline for raising complaints
o clauses that change usages of trade or past courses of dealing
EXAMPLE: Buyer sends Seller a purchase order for 1,000 widgets at the advertised
price of $10 each. Seller sends Buyer an Acknowledgment of Order that promises
delivery of the widgets at the stated price and includes boilerplate language that negates
all warranties and requires payment for the order to be complete within 30 days of
delivery, which is standard in the widget trade.
 “Different” terms in the two writings dealing with the same topic knock each other out (the “knockout”
rule).
EXAMPLE: Buyer’s purchase order contains a choice of law provision stating that California law will
govern disputes arising from the transaction, and Seller’s order acknowledgment states that New York
law will govern. Under the majority rule, neither provision is part of the parties’ contract, and if the
parties desire a choice of law provision, they will have to negotiate one from scratch

Notes:

8
MBE CONTRACTS
 Conditional Acceptance
o If the offeree’s “acceptance” is specifically conditioned on offeror first agreeing to the additional terms
in the acceptance before offeree will proceed, this nonconforming, conditional acceptance will NOT
be effective to form a K (i.e., it’s a counteroffer).
 No K is formed by the writings until the offeror expressly assents to the additional terms.
 Contracts Formed by Conduct
o The UCC provides that the parties’ conduct in recognizing the existence of a contract is sufficient
to establish a contract even though their writings do not otherwise establish a contract.
EXAMPLE: Buyer sends order to Seller, Seller responds with a conditional acceptance containing a
negation of warranties, and the parties have no further communications. Seller nevertheless ships
the ordered goods, and Buyer accepts and pays for them. Although the parties’ writings do not form
a contract—because Seller sent a conditional acceptance—a binding contract is formed by their
conduct.
o The terms of the contract will be:
 Terms on which the writings of the parties agree; and
 Default terms provided by the UCC
Note: express terms in the parties’ communications which do not match or agree are omitted.

Notes:

9
MBE CONTRACTS
CONSIDERATION
1) General Rule = Promise is unenforceable unless it is supported by consideration
 Bargain Theory = Promise is supported by consideration if based on a bargained-for exchange 
there was something (goods or services) that was promised, and the promise must have been made in
order to obtain something else of value (quid pro quo for making the promise – usually a return promise
or performance)

2) Benefit / Detriment Test (legal value analysis) = Whether there is a benefit to the promisor and/or a
detriment to the promise.
 Legal Detriment Test = Whether promisee is:
o doing something he had a legal right NOT to do, or
o forgoing some activity he HAD a legal right to do
EXAMPLE: Promoter promised musician $5,000 to play a concert at Carnegie Hall. Promoter then
tried to back out of the promise, claiming that “you would have done it for free.”
EXAMPLE: Uncle promised nephew $5,000 if nephew would give up smoking and drinking. The
uncle’s estate, hoping to avoid this obligation, argued that the nephew didn’t incur a detriment
because giving up smoking and drinking was good for his health.

3) Inadequacy of Consideration: courts don’t police the equivalence or fairness of the exchange

4) Illusory Promises: Promise of performance that leaves performance to the unlimited discretion of the
promising party  does NOT constitute consideration
EXAMPLE: You agree to paint my portrait, and I agree to pay you $1,000 if I decide I want it.

5) Gratuitous Promises: Promise to make a gift, generally unenforceable due to insufficient


consideration
 Exception: Gratuitous transfers are legally binding
EXAMPLE: If I promise you a new car for your birthday, that would constitute a gratuitous promise and
would be unenforceable if I decline to follow through. However, if I give you a new car for your birthday,
then the transfer is legally binding, and I cannot later change my mind and take back the car.

Notes:

10
MBE CONTRACTS

Alternatives in the Absence of Consideration


6) Past or Moral Consideration
 General Rule: A promise in exchange for something already given or performed is NOT supported by
consideration.
EXAMPLE: In Mills v. Wyman [20 Mass (3 Pick.) 207 (1825)], the court refused to enforce the
father’s after-the-fact promise to compensate a Good Samaritan for nursing his dying son, and the
case would likely come out the same way today.
 Exceptions (these are enforceable):
o A written promise to pay a debt barred by limitations;
o A written promise to pay a debt discharged by bankruptcy;
 Material Benefit Test (minority rule): promise made in recognition of a past benefit conferred is
enforceable if:
o Promisee conferred the benefit on the promisor (not a third party); and
o The benefit is material
EXAMPLE: A sees that B is in grave danger and heroically intervenes to save the latter, injuring
himself in the process. B gratefully promises to compensate A for his efforts. These are the facts of
Webb v. McGowin [27 Ala. App. 82 (1935)], where the court enforced the promise because A’s
efforts bestowed a material benefit (the saving of a life) on B.
EXAMPLE: Because the Good Samaritan in Mills v. Wyman bestowed the benefit of nursing
services on the promisor’s son rather than on the promisor, the promise would not be enforceable.

Notes:

11
MBE CONTRACTS
7) Promissory Estoppel: promisee that reasonably relies to his detriment on gratuitous promise may be
able to enforce that promise even without consideration. 4 Requirements:
 A promise
EXAMPLE: Statements like “I’d like to give you some money to help you with college” would be
considered too vague to be an actual promise. Alternately, “I’ll give $25,000 toward your tuition this fall”
would be specific enough to qualify as a promise.

 Foreseeable reliance
EXAMPLE: The widow’s move to her brother-in-law’s farm on the faith of his promise would have been
reasonably foreseeable to the brother-in-law at the time he made her the promise. However, if the
widow had purchased a high-end SUV to facilitate the move, this would likely not have been
foreseeable.

 Actual reliance (must be induced by the promise)


EXAMPLE: If the widow had already decided to make the 60-mile move before her brother made her
the promise, then this would not be actual reliance.

 Injustice without enforcement


o Factors to analyze “injustice” requirement:
 Strength of proof of the other three requirements;
 Blameworthiness or willfulness of the breach;
 Relative position or equities of the parties;
 Extent to which the reliance was detrimental; and
 Availability of alternatives short of enforcing the promise

HYPOTHETICAL: Uncle promises his nephew $8,000 to enable the latter to purchase a neighbor’s car for
transportation to school and an after-school job. In reliance on the promise, the nephew secures possession of
the vehicle and promises the neighbor the $8,000. If the uncle reneges on his promise to pay, is promissory
estoppel available if the neighbor is willing to accept the return of the car and release the nephew from debt?
What if the car has a market value far in excess of $8,000, enabling the nephew to sell the car, cover the debt
to his neighbor, and recoup any costs associated with the sale?

Notes:

12
MBE CONTRACTS
STATUTE OF FRAUDS (SOF)
1) General Rule = Oral and written Ks are equally enforceable
 SOF Exception = If the K falls under the SOF  It must be in writing + signed by party against whom
enforcement is sought
2) Categories of Contracts Subject to SOF: 6 categories governed by the SOF = MYLEGS
 Marriage Ks
 Ks that can’t be performed within one Year of their making
 Ks for the sale of Land
 Ks of an Executor or administrator to answer for a duty of a decedent
 Ks of Guarantee or suretyship
 Ks for the Sale of goods at a price of $500 or more (UCC 2-201)
3) Some categories have specific limitations within them that narrow the reach of the Statute of Frauds.
4) Contracts Not to Be Performed within a Year
 Measured from date K made, not date performance begins
EXAMPLE: In June of her first year of law school, Law Student enters an oral agreement with Law Firm
to work for the firm during June, July, and August of her second summer. Although the duration of the
contemplated performance is only three months, the performance will not be complete until 14 months
after the making of the agreement. Accordingly, the contract is governed by the one-year provision and
a signed writing is required in order to secure enforcement.

 At point of formation, is it at all possible (even if not probable) to complete required performance within
1 year?
EXAMPLE: It is highly unlikely that the construction of an oil-producing facility in a war-ravaged country
would be completed within a year. But because the performance is possible within that time—even if
exceedingly unlikely—the agreement is not governed by the one-year provision and no signed writing is
required.
EXAMPLE: One of the parties to an oral construction agreement attempts to back out 14 months into
the project and raises the Statute of Frauds as a defense. Because the prospect of performance is
measured from the point of making the contract (when completion within a year was possible) rather
than from the time of the dispute (when completion within the first year is clearly no longer possible),
the agreement is not governed by the one-year provision and no signed writing is required.
EXAMPLE: A three-year exclusive representation agreement between a professional athlete and his
agent cannot be performed within a year—no matter how diligently the agent works. Accordingly, the
agreement is governed by the one-year provision and a signed writing is required.

Notes:

13
MBE CONTRACTS
 Frequently Tested Situations
o K that can be breached or excused within a year of its formation
 This is potentially true of any K and so would swallow the one-year rule
 Irrelevant. What matters is not whether the K can be breached within one year but whether the
K can be performed in full by its terms within one year of its formation.
o A lifetime or permanent contract of employment is not governed by the one-year rule because
employee’s death is possible within a year

5) Guaranty/Suretyship Agreements – Main Purpose Exception:


Although guarantee contracts (secondary obligation) are generally within the Statute of Frauds, a primary
exception is the main purpose doctrine. That is, where the main purpose of the guarantor is to protect
his own economic interests, the guarantee agreement is not governed by the Statute of Frauds, and
can be oral.

HYPOTHETICAL: Joe Plumbing Co. secures a loan from Piggy Bank. N. Vestor, who owns a controlling interest in
the plumbing firm, makes an oral promise to the Bank to guarantee repayment of the loan. Is the guarantee within the
Statute of Frauds? What if Vestor is a shareholder in Joe Plumbing Co. but does not own a controlling interest in the
firm?

Notes:

14
MBE CONTRACTS
6) Satisfying the Statute of Frauds
 Writing Requirement: All that is necessary is that the writing be a memorandum of the agreement
which can be prepared before, during, or after formation
EXAMPLE: “Dear Tony, I regret to inform you that I cannot go through with the sale of my racehorse to
you. Although the $100,000 you agreed to pay is very generous, my accountant has advised me of the
tax implications, which I had not considered, and so I must regretfully bow out.”
o Following terms are required:
 Identity of the parties to the transaction;
 The nature and subject matter of the K; and
 The essential terms of the agreement, such as price and date for performance

 Signature: Any symbol with intention to authenticate the writing (ex: initials, typed, stamped or
preprinted signature, letterhead)

 Performance: SOF may be satisfied with respect to some categories of governed contracts via part
performance, in an action for specific performance.
o Land sale Ks: Part performance requires a showing of any combination, or all three of the
following:
 Payment of all or part of purchase price
 Taking possession of land
 Making substantial improvements to property
o One-Year Ks:
 If fully performed enforceable despite SOF
 If only partially performed not enforceable

Notes:

15
MBE CONTRACTS
1) UCC Statute of Frauds
 Does the agreement fall within the SOF? Ks for sale of goods for price of $500 or more are under the
UCC SOF.
 5 ways to satisfy UCC SOF
o Signed Writing:
 Writing
 Quantity
 Signed by the party against whom enforcement is sought

o Merchant’s Confirmation: Two merchants enter oral agreement, 1 sends the other written
confirmation of agreement. SOF satisfied against the recipient merchant if:
 Both sender and recipient are merchants
 Writing is in confirmation of the K and contains a quantity; and
 Recipient does not send written objection within 10 days

EXAMPLE: A and B enter an oral agreement under the terms of which A is to sell B five
tobacco barns for a total of $5,000. The following day, B sends A a written confirmation of the
agreement and two days later B receives a signed reply that reads in pertinent part: “My
understanding is that we don’t have a contract between us, for as I mentioned I am still
entertaining other offers for the barns. My apologies for any confusion that may have been
caused.” Because A’s reply objects to the confirmation’s contents, the confirmation does not
satisfy the UCC Statute of Frauds against A.

Notes:

16
MBE CONTRACTS
o Judicial Admission: Party admits K formation in pleading, testimony, or otherwise in court.

o Partial Performance: Despite absence of writing, an otherwise valid K is enforceable for:


 Goods for which payment made/accepted
 Goods which have been received/accepted

o Specially Manufactured Goods:


 SOF satisfied against buyer who orders custom goods from a manufacturer if:
 Manufacturer detrimentally relied by beginning performance before buyer’s withdrawal; and
 Manufacturer can’t resell the goods in the ordinary course of business

Notes:

17
MBE CONTRACTS
GAP-FILLERS, INTERPRETATION, AND THE PAROL EVIDENCE RULE
1) UCC Default Provisions: used to fill in the gaps when K doesn’t address the matter, but parties free
to set default rules aside by contracting otherwise
 Implied Warranties: (3)
o Warranty of Title:
 For the sale of all goods, there is an implied warranty of:
 Good title to the goods;
 Rightful transfer of the goods; and
 No liens or security interests are attached to the goods
 Can only be excluded or modified by:
 Specific language; OR
 Circumstances which give buyer reason to suspect seller does not claim unencumbered title
o Warranty of Merchantability:
 Goods fit for ordinary purposes for which those goods would be used
 ONLY applies if seller is a merchant
 Displaced by:
 Specific use of word “merchantability” and conspicuousness if in writing; OR
 Any other language or circumstances that would be reasonably understood by a buyer to
exclude the warranty (“as is” or patent defects)
o Warranty of Fitness for a Particular Purpose:
 Goods being sold are fit for the particular purpose buyer intends to use them for
 ONLY applies when, at time of contracting, seller has reason to know:
 Particular purpose for which the goods are required AND
 The buyer is relying on seller’s expertise to select reasonable goods
 Warranty negated when:
 Disclaimer is written, clear, and conspicuous; OR
 Goods have patent defects which were easily detectable

Notes:

18
MBE CONTRACTS
 Express Warranties:
o Goods will conform to some standard, arises whenever seller expressly makes them as part of the
basis of the bargain in the following ways:
 Any affirmation of promise or fact;
 Any description of the goods; or
 Any sample or model;
o Need not use words “warrant” or “guarantee” to create. However, vague or “sales talk” statements
are considered “puffing,” not warranty.

 Missing Terms: Under the UCC, where there are pertinent missing terms, the following default
rules will fill in those missing terms:
o Price term = Reasonable price at time established by the K for delivery
o Time term = Reasonable time
o Place of delivery term = Seller’s place of business (i.e. FOB seller)

2) Common Law Default Rules for Service Contracts


 Missing price term: if one party performs services at the request of another, but no price is discussed
in advance, then CL default rule applies  reasonable value for services rendered
3) UCC/Common Law Obligation of Good Faith and Fair Dealing
 Another source of “gap-filling” is the obligation of good faith and fair dealing in the performance and
enforcement of the K. (UCC and Restatement)
 Good faith = honesty in fact and the observance of reasonable commercial standards of fair dealing in
the trade
 Good faith obligation where terms of K leave critical term, such as price, satisfaction, or quantity open
to the determination of one party.
o Open price terms = If K leaves price to be fixed by one of the parties, then that party must fix the
price in good faith
o Satisfaction terms = If K contains a satisfaction clause or similar term, then the determination as to
whether a party’s performance obligation must be exercised in good faith
EXAMPLE: A commercial developer and the seller of a piece of land enter into a contract. The
developer places a condition on his purchase of the land that the seller first acquires “satisfactory
leases” for the future occupancy of the land. The developer must determine in good faith whether or
not the future leases are “satisfactory.”

Notes:

19
MBE CONTRACTS
o Open Quantity Terms:
 Arises in two contexts
 Output contract: Buyer agrees to purchase all or a percentage of a seller’s output for a
particular good
 Requirements contract: Seller agrees to supply the buyer with all or a percentage of the
buyer’s requirements for a particular good
 Under the UCC, the party entitled to determine the particular quantity of goods to be sold –
either the buyer demanding delivery of his requirements or the seller demanding purchase of
her output – must make that determination in good faith.
 UCC also prohibits any unreasonably disproportionate demand or tender, if there was either:
 A stated estimate; OR
 A past course of dealing
EXAMPLE: Seller and Buyer are parties to a three-year contract obligating Seller to supply
Buyer with the latter’s upsidasium bearing requirements for Buyer’s turbojet engine
manufacturing plant. The market price for upsidasium suddenly skyrockets, and Buyer
greatly increases its demand for bearings with the intention of selling them at great profit to
third parties. Seller is not obligated to fill the order because Buyer’s requirements demand
was made in bad faith.

EXAMPLE: The upsidasium market remains stable, but because of a decline in the
commercial aviation industry, the demand for Buyer’s engines declines precipitously and
accordingly Buyer reduces its monthly demand for bearings by 90% in comparison with the
previous 24 months. Although Buyer’s reduced demand was made in good faith, it is
unreasonably disproportionate to its prior requirements and accordingly violates Section 2-
306.

Notes:

20
MBE CONTRACTS
INTERPRETING AMBIGUOUS LANGUAGE

1) Objective Meaning Trumps Subjective Meaning


EXAMPLE: Contractor and Homeowner enter a contract for renovations to Homeowner’s home. The
contract incorporates by reference “the specifications dated 1/11/07,” which had been prepared by
Homeowner and were attached to the contract before signing. Contractor mistakenly thinks that the
specifications referenced were an earlier version prepared by Contractor, and he signs the contract on
that understanding. The specifications prepared by Homeowner and referenced in the contract are
binding on both parties.

 Exception to the rule:


o When one party has reason to know of other party’s subjective understanding the first party is
bound by that meaning;
EXAMPLE: The facts being otherwise as stated in the previous example, just before the parties
sign the contract, Homeowner overhears Contractor tell his business partner that he was glad
Homeowner had agreed to Contractor’s version of the specifications. Because Homeowner knew of
Contractor’s understanding of the contractual terms, that understanding controls.

2) Contra proferenem:
 Rule: If ambiguous term is included in K, then in case of doubt it is construed against the drafter
 Contracts subject to this rule of construction:
o All Ks drafted by one of the parties, whether an adhesion K or one between parties of equal
bargaining power

Notes:

21
MBE CONTRACTS
TRADE USAGE, COURSE OF DEALING, COURSE OF PERFORMANCE

1) Extrinsic evidence that can be used to fill gaps and resolve ambiguities in Ks
 Definitions under UCC:
o Usage of trade is any practice or method of dealing having such regularity of observance in a place or
trade as to justify an expectation that it will be observed with respect to the transaction in question [what
all members of the trade do]
o Course of dealing is a pattern of conduct concerning previous transactions between the parties
that is fairly to be regarded as establishing a common basis of understanding for interpreting their
subsequent expressions and other conduct [what the parties did in their previous contracts].
o Course of performance is present when a particular contract involves repeated occasions for performance by
a party and the other party, with knowledge of the nature of the performance and opportunity for objection to
it, accepts the performance or acquiesces in it without objection [how the parties have acted under their
current contract].
 Admissible uses: To fill in gaps (supplement terms) + resolve ambiguities (explain terms)
 Inadmissible uses: To contradict express terms of K
 If conflict, course of performance prevails over course of dealing and usage of trade, and course of
dealing prevails over usage of trade
EXAMPLE: Trade Usage Can Fill in a Gap in the Terms of the Contract: A is one of many suppliers
of upsidasium, and B is one of A’s long-standing customers. A fills an order by B and demands
immediate payment; B refuses. It is standard practice in the upsidasium industry for purchasers to pay
invoices upon delivery, and therefore B’s refusal to do so is in violation of the parties’ contract.
EXAMPLE: Course of Dealing Trumps Trade Usage: Same facts as the previous example except
that in the past, A has regularly given B 30 days to pay for deliveries. Because this course of dealing
between the parties trumps the payment-upon-delivery usage of trade, B has 30 days within which to
make the required payment.
EXAMPLE: Express Terms of the Contract Trump Both Course of Dealing and Trade Usage:
Same facts as the previous example except that, the parties’ contract contains a provision requiring
payment within seven days of delivery. Because the express terms of a contract override both course of
dealing and usage of trade, B has seven days within which to make payment despite the past practice
of paying in 30 days and the industry practice of immediate payment.

Notes:

22
MBE CONTRACTS
THE PAROL EVIDENCE RULE

1) Governs admissibility of oral and documentary evidence of negotiations and other communications
between the parties that took place prior to or contemporaneously with the execution of the K

2) Integration
 Partial integration = Terms within K are intended as final expression of those specific terms
o Note: the terms “full integration” and “partial integration” are interchangeable.
 Complete integration = K intended to represent a complete and exclusive statement of all the terms
 Determining Partial and Complete Integration
o Merger clause = Writing contains complete/entire agreement or other words to that effect

3) Purpose for Which Evidence is Introduced:


 Effect of the rule depends on the purpose the parol evidence is being introduced
 Purpose #1: To Explain or Interpret Terms of Written Contract
o Majority rule = parol evidence is always admissible for this purpose
EXAMPLE: A and B have a written contract under the terms of which A agrees to sell B “all the
uncut timber on my property.” Evidence of conversations between the parties during a visit to the
timber site is admissible to explain that “property” means A’s country cottage and not A’s home in
the suburbs.
 Purpose #2: To Supplement Terms of Written Contract
o Rule: parol evidence is admissible for this purpose unless the K is completely integrated (i.e., has
merger clause)
EXAMPLE: A and B had a written contract under the terms of which A agrees to sell B “all the
uncut timber on my property,” and the contract contained detailed terms with respect to the felling
and transportation of the timber as well as a merger clause stating that the contract was “a
complete and exhaustive account of the obligations between the parties.” Evidence of an oral
agreement between the parties at the time of signing the contract that A would pay to have the
trees inspected for termites before cutting would be admissible to supplement the agreement only if
the court concluded that the contract was partially (rather than completely) integrated. Some courts
would consider the merger clause to be conclusive on the question of complete integration and thus
bar the evidence, but other courts would treat it as persuasive only, evaluate the proffered evidence
of the supplemental oral agreement, and admit it if they concluded that the parties did not intend the
previously drafted merger clause to bar it despite their subsequent signatures on the contract.

Notes:

23
MBE CONTRACTS

o UCC distinction: Trade usage, course of dealings, and course of performance can supplement a
completely integrated agreement
EXAMPLE: Same facts as in the previous example, except A and B are logging companies and B
offers evidence that sellers doing business in the commercial lumber trade invariably pay to have
the trees inspected for termites before cutting. This evidence would be admissible even if the court
concluded that the contract was completely integrated because under the UCC uncut timber equals
“goods” and usage of trade evidence is admissible to supplement a written agreement.

 Purpose #3: To Contradict Terms of Written Contract


o Rule: parol evidence is not admissible for this purpose
EXAMPLE: A and B have a written contract under the terms of which A agrees to sell B “all the
uncut timber on my property.” Evidence of an exchange of letters between the parties in which they
agreed that their agreement would not include the shady trees in the immediate vicinity of A’s
country cottage would contradict the written expression—i.e., that A had agreed to sell B “all the
uncut timber on my property”—and would therefore be admissible only if the court concluded that
the quoted provision not intended by the parties to be the final word with respect to which timber
would be cut (i.e., that the writing was not integrated). In making that determination, the court is free
to consider the exchange of letters and other extrinsic evidence.
4) When PE Rule Will Not Apply
 Subsequent Agreements
EXAMPLE: Buyer and Seller enter into a written agreement for the sale of 1,000 widgets. Two months
before delivery is to be made, the parties orally agree to modify the agreement and specify the goods
as nonstandard type-Y widgets, with Buyer agreeing to pay an additional cost. Evidence of this
modification would not fall within the parol evidence rule, as it was made subsequent to the execution of
the written agreement.

 Collateral Agreements: PE rule will not affect agreements between parties that are entirely
distinct from the written agreement of the K at issue
EXAMPLE: A and B execute a detailed written contract, complete with a merger clause, by which A
agrees to sell A’s car to B. As part of the transaction, the parties orally agree that B may park the
automobile in A’s garage for one year, paying $25 per month. Despite the completely integrated writing,
either A or B may adduce evidence of the parking arrangement under the collateral agreement rule.

Notes:

24
MBE CONTRACTS
 Attack on Validity of the Written Agreement
o PE rule only applies if there’s a valid written agreement. Thus, the PE rule will not bar efforts to
prove the following bases for the invalidity of the K:
 Failure of oral condition precedent to agreement
EXAMPLE: The parties to a written contract for the sale of artwork orally agree that the sale will
not take effect unless and until a certificate of authenticity is issued by a named art expert.
Because authentication is a condition precedent to the sale, the parol evidence rule will not bar
proof of the oral agreement.

 Mistake or duress;

 Fraud;

 Reformation: for plaintiff to obtain reformation, it must be shown:


 There was an antecedent valid agreement;
 That this antecedent agreement is incorrectly reflected in the writing because of mistake or
fraud; AND
 That proof is established by clear/convincing evidence

Notes:

25
MBE CONTRACTS
PERFORMANCE, MODIFICATION, AND EXCUSE

1) Obligations under the UCC – Risk of Loss Rules


 Seller’s obligations = transfer and deliver goods
 Buyer’s obligation = accept and pay for goods
 When something happens to the goods before the buyer receives them:
o If seller bears ROL, he must provide replacement goods
o If buyer bears ROL, he must pay the K price regardless

 Carrier Cases versus Non-Carrier cases:


o Carrier cases: parties to a sale of goods K agree to use common carrier (trucking firm, train, etc.)

o Non-carrier cases: parties to a sale of goods K do not agree to use a common carrier

o Non-carrier cases - When does ROL transfer to buyer?


 If seller is NOT a merchant
 As soon as goods are made available to him (tendered)
 If seller IS a merchant
 Once goods are physically in buyer’s possession

o Carrier Cases – When does ROL transfer?


 Shipment contract: ROL passes to buyer when goods are delivered to the carrier
 Identified as “FOB Seller,” or K is silent (default rule)
 Buyer bears ROL if goods are damaged in transit
 Destination contract: ROL passes to buyer when goods are tendered at the destination point
specified in the K
 Identified as “FOB Buyer”
 Seller bears ROL if goods are damaged in transit

Notes:

26
MBE CONTRACTS
1) Modification
 Common Law
o Preexisting Duty Rule: a promise to increase compensation for duties already owed is
unenforceable because there is no consideration for the modification
EXAMPLE: The captain of a fishing vessel promised to pay $100 to each of the sailors for their
work on a fishing voyage. Midway through the voyage, the sailors threatened to cease work unless
they were promised an additional $50 each, and the captain reluctantly agreed. At the end of the
voyage, the captain paid each of them the originally promised $100 but refused to pay the $50
increase, so the sailors sued to recover the additional amount. Because the sailors were already
obliged to perform the work in question under the terms of their original contract with the captain,
his promise of an additional $50 is unenforceable under the pre-existing duty rule.
o Exceptions:
 Mutual Modification of an existing K is enforceable if both parties agree to different
performance from what was originally required by the original K
EXAMPLE: During the course of the fishing voyage, the ship’s cook takes ill, and the captain
instructs the youngest of the sailors to perform the cook’s duties in addition to his fishing duties.
The sailor refuses to perform the work unless the captain promises him an additional $50 in
compensation, and the captain reluctantly agrees. The promise of additional compensation is
enforceable as a “mutual modification” of the original contract.
 Unforeseen Circumstances: Preexisting duty rule not applicable if increased compensation is
given in exchange for a promised performance that has been rendered substantially more
burdensome than reasonably anticipated when K formed
EXAMPLE: The fishing nets provided by the captain turn out to be defective and thus increase
the workload of the sailors in a manner substantially in excess of what was reasonably
contemplated under the original contract. The sailors threaten to cease the fishing unless they
are promised an additional $50 each, and the captain reluctantly agrees. The promise is
enforceable despite the pre-existing duty rule in view of circumstances not reasonably
anticipated by the parties at the time of contracting.

Notes:

27
MBE CONTRACTS
 UCC: Agreement modifying an existing K needs no consideration to be enforceable as long as made in
good faith
EXAMPLE: A manufactures upsidaisium bearings and has a three-year contract to supply B with the
bearings the latter uses to make turbo-jet engines for commercial aircraft. Because of an unexpected
world shortage of upsidaisium, the price of the metal A uses to manufacture the bearings triples over a
short period, and A advises B that it can’t continue supplying the bearings unless B agrees to increase
the price to the point that A will break even on the supply contract. B reluctantly agrees. Is the
modification enforceable?

2) Excusing Performance due to Faulty Assumptions


 A party entering a contract makes many assumptions about the present and future. When such assumptions turn
out to be faulty, the parties may be excused under the doctrines of mistake, impossibility, impracticability, and
frustration of purpose.
 Faulty Assumptions Regarding Present Material Facts: Mistake
o Unilateral Mistake – 1 party’s mistake about present material fact(s) is NOT excused unless other
party knew/had reason to know of the party’s mistake
EXAMPLE: A agrees to sell B a cow, which A knows to be barren, and, as the parties are writing up
and signing the agreement, B asks A a series of questions about the care and feeding of pregnant
cows. Because A has reason to know that B is mistaken with respect to the cow’s capacity to bear
calves, B’s obligation to purchase the cow is excused on account of unilateral mistake.

o Mutual Mistake – Voidable by disadvantaged party when all 3 met:


 Mistaken assumption relates to material facts (not just value);
 Mistake made by both parties; and
 Disadvantaged party did not bear risk of mistake under the K
EXAMPLE: A agrees to sell B a cow at beef cow prices because at the time of contracting, both
parties were under the assumption that the cow was barren. A short time later, the cow was
discovered to be with calf, which greatly increased her resale value. The contract is voidable at
the option of A.
EXAMPLE: A agrees to sell B a cow at beef cow prices. At the time of contracting, both parties
share the mistaken impression that the cow is barren. B tells A that he is still going to try to
breed the cow in any case. In this case, A bears the risk of B’s efforts succeeding, because if B
succeeds, it will prove the parties’ assumption that the cow was barren wrong. So A is
accordingly bound by the contract even if the cow turns out to be fertile.

Notes:

28
MBE CONTRACTS
3) Faulty Assumptions Regarding Future Facts: Impossibility, Impracticability, and Frustration of
Purpose
 Impossibility: Both parties excused if performance has been rendered impossible by events occurring
after K formed.
o Requirement #1: Impossibility must be objective
 Objective impossibility occurs when performance literally impossible for anyone due to
circumstances beyond control of the parties
EXAMPLE: X promises to sell Y his horse, but the horse dies before X can deliver the horse.
 Subjective impossibility occurs when performance becomes impossible because of some failure
or fault on the part of the performing party
EXAMPLE: A party fails to have enough money to either make a promised payment or to obtain
the components required for the production of a promised product.

 In what circumstances are we likely to find objective impossibility?


 Circumstance #1: when the subject matter of the K is destroyed
EXAMPLE: Buyer promises to buy Farmer’s 2009 wheat crop, the entirety of which is
destroyed just before harvest by wheat blight. The Farmer’s performance is excused on the
basis of impossibility.

 Circumstance #2: when there is a personal services K and the performing party has died or
become incapacitated

 Circumstance #3: when supervening law or legal developments have rendered performance
legally impermissible

o Requirement #2: The contingency that creates the impossibility was not known to the parties at the
time of making the K; it arose after the K and was unanticipated.

Notes:

29
MBE CONTRACTS
 Impracticability: Courts are reluctant to excuse performance for any reason other than
impossibility. However, under the doctrine of impracticability, a promisor may be excused from
performance where the following two elements are proven:
o Contingency causing impracticability was unforeseen; and
o Increased cost / burden of performance would be far beyond what either party anticipated

 Impracticability under UCC:


o Increased costs, and the rise or collapse of a market are viewed as business risks, the sorts of
contingencies that fixed-price contracts are expected to account for.
o UCC cases where impracticability has been found typically involve shortages caused by war or
embargo, local crop failure, or unforeseen shutdown of major sources of supply (e.g. natural
disaster).

 Frustration of Purpose: Where a contingency occurs that dramatically reduces the value of
performance to the receiving party.
EXAMPLE: The doctrine of frustration of contractual purpose has as its source the famous case of
Krell v. Henry, which involved the owner of a London flat with a “ringside” view of the forthcoming
coronation parade who agreed to lease the flat at premium prices to a lessee eager to witness the
festivities. The parade was canceled when the King became ill, and the lessee’s contractual obligations
to the owner were excused on the ground that going through with the rental agreement in the absence
of its raison d’être “cannot reasonably be said to have been in the contemplation of the parties at the
date of the contract.”

o Modern Test: (3 requirements)


 Principal purpose in entering K is substantially frustrated;
 This means that the frustration of incidental or non-material purposes would not trigger the
excuse of frustration of purpose
 Frustration was substantial in nature; and
EXAMPLE: In a variation on Krell v. Henry, where the King did not become ill and the parade
was not cancelled, the town erected bleachers that only partially obstruct the parade view from
the apartment.
 Non-occurrence of the event that caused frustration was a basic assumption of the K

NOTE: Impossibility, impracticability, and frustration of purpose are available under common law and UCC.

Notes:

30
MBE CONTRACTS

31
MBE CONTRACTS
4) Excusing Performance by Agreement of the Parties
 Rescission: Is permissible where both parties to a K have remaining performance due. Consideration
is provided by each party’s discharge of the other’s duties.
 Accord and Satisfaction: The parties may make an accord, which is permissible when one party has
already performed and is waiting on the other party to finish performing.
o Accord = obligee promises to accept substituted performance in satisfaction of the obligor’s
existing duty
EXAMPLE: Purchaser has paid builder in full for the construction of a house. Because of difficulties
that neither party to the contract could foresee, the builder is only able to partially complete the
construction project. Purchaser agrees to a lesser performance in return for a partial re-payment.
This agreement is an accord. Builder then completes the agreed-to substituted performance. This
performance amounts to satisfaction of the accord.

o Legal Effects:
 Accord and satisfaction satisfies the obligor’s original obligation
 The accord alone does not discharge the obligor’s duty; it only suspends the obligor’s duty.
o Validity of Accord – Consideration Required
 General contract law applies – consideration is required
 May be sufficient consideration if:
 Where the accord involves an agreement for partial or substituted performance, the
substituted performance differs significantly from the original performance, or its obligation is
doubtful
 Where the accord involves an agreement for partial payment, there is a good-faith or bona
fide doubt about the amount owed

Notes:

32
MBE CONTRACTS
5) Anticipatory Repudiation (AR)
 AR may be established by:
o Party’s definitive statement that it will breach, or
o Party’s voluntary act that renders party unable to perform its contractual obligations

 Adequate Assurance of Performance


o If AR cannot be established but there are reasonable grounds for insecurity, the insecure party
may make a demand for adequate assurance of performance
EXAMPLE: After making a contract with a manufacturer. A wholesaler become aware that the
manufacturer’s employees are out on strike. This gives reasonable grounds to confirm with the
manufacturer that the contract will be performed.

 Failure to Give Adequate Assurance


o Failure to respond with reasonable assurances = repudiation. This can occur where the other party:
 Does not respond to the demand for assurance in a reasonable time (30 days under the UCC),
or
 Does not respond in a way that provides reasonable assurances

 Rights of Aggrieved Party upon Repudiation


o Cancel K and terminate all rights/obligations under it;
o Bring action for damages or specific performance; OR
o Ignore the repudiation and continue under the K

 Retraction of a Repudiation
o A party who has made an AR to the other party may retract unless/until the other party:
 Acts in reliance on repudiation;
 Accepts repudiation by signaling this to breaching party; OR
 Commences suit for damages/specific performance

Notes:

33
MBE CONTRACTS
6) Conditions
 In some contracts, the obligation to perform is conditioned upon some event or action by the other
party. Where there are such conditions in a contract, obligations are triggered when that conditioning
event or action occurs.
 Rules Governing the Failure of a Condition
o Under Common Law: the rights of the parties in the event of a failed condition depend on whether
the condition is express or implied.
 Failure of Express Condition: any failure of an express condition (i.e., less than 100% strict
compliance) will discharge the party’s obligation to perform
 Identifying Express Conditions: look for clear language of the parties
EXAMPLE: “The party’s duty to perform is expressly conditioned on the promised
performance.”
EXAMPLE: “There is no obligation to proceed unless and until the following steps are taken.”
EXAMPLE: “The party has a duty to perform on the condition that certain events occur” or, “only
if certain event occur.”
EXAMPLE: “Payment is due upon completion”
OTHER CONDITIONAL PHRASES: “on condition that,” as long as,” “when,” “provided that”
 Excusing Failed Express Conditions:
 Two situations in which failure of the condition may be excused such that the performance
obligation of the party who stood to benefit from the condition is not discharged
 Situation #1: Waiver
o The party who has been discharged from performing by the failed condition may waive
the right to discharge and perform anyway
NOTE: Some courts treat this as a mid-term modification, which in a common law case
requires consideration.
 Situation #2: Bad Faith Conduct
o When a party acts in bad-faith so as to prevent the condition, that party’s
performance obligation will not be discharged
EXAMPLE: In a contract for the sale of real property, the contract is conditioned on
the buyer securing financing. If the buyer fails to apply for financing, this would be
considered conduct in bad faith, and the buyer would be obligated to pay.

Notes:

34
MBE CONTRACTS

 Implied Conditions (Material Breach and Substantial Performance)


 When the possibility of a breach is not addressed by an express condition, it is still a breach;
however, under the law of implied conditions, courts can treat that breach in one of two
ways: either as a material breach or as substantial performance.
 Material Breach versus Substantial Performance
o Material Breach: If breach is serious enough, court will treat like failure of an express
condition. The aggrieved party is generally discharged from his own performance obligations
o Substantial Performance: If the breach is less serious, court will treat performance as
“close enough,” party has rendered substantial performance. The aggrieved party will
not be discharged of his own performance obligations

HYPOTHETICAL: Contractor builds a house for Landowner, and—unbeknownst to Contractor—the plumbing


subcontractor installs a brand of pipe that differs from the brand specified in the contract but is in every
important respect the same quality. It would be an extremely expensive and burdensome task for Contractor to
remove and replace the pipe. Is this a case of substantial performance or material breach?

HYPOTHETICAL: Contractor builds a house for the Yuppies, a young couple who are foodies and love to
entertain, and accordingly their plans include a spacious kitchen. The Yuppies spend the year in Paris while
the house is under construction, and in their absence and without prior consultation, Contractor decides to
deviate from the plans and build a smaller kitchen, thus enabling Contractor to save significant construction
costs. Is this a case of substantial performance or material breach?

Notes:

35
MBE CONTRACTS
 Failed Condition that Cannot Be Excused
 Where a condition has failed and cannot be excused, there are other methods of
enforcement to mitigate the consequences for the breaching party

 Divisibility of the Contract


o Breaching party can argue K is divisible and only part of K has been materially
breached
o Divisibility Test = is it easily apportioned into agreed equivalents?
EXAMPLE: A one-year cleaning service contract would be divisible if the services were
provided and paid for on a monthly basis, as it would be easy to apportion the larger
exchange into “pairs of part performance” that are “properly regarded as agreed
equivalents”—i.e., a month’s worth of cleaning in exchange for a month’s pay. To put it
another way, the one-year contract is merely the sum of the 12 monthly exchanges.
EXAMPLE: A enters into a contract to build a house for B for $100,000, with progress
payments of $5,000 due in monthly installments during construction and a balloon payment
upon the architect’s certificate of satisfactory completion. The contract is not divisible, for the
performances and the progress payments are not “agreed equivalents,” and the whole is
greater than the sum of the parts, since no reasonable person would enter a free-standing
contract for a month of construction work.

TIP: If the whole is merely the sum of its parts, then it’s divisible. But if it’s more than the sum
of the parts, it is not divisible (i.e., a house)

 Quantum Meruit: Where a party failed to fulfill an express condition or is in material breach,
breaching party may still recover in quantum meruit:
o Reasonable value of benefits conferred
o However, recovery will be reduced by damages caused by breach

Notes:

36
MBE CONTRACTS

 Failure of Condition under UCC


o Express and Implied Conditions in Relation to the Perfect Tender Rule
 Under the perfect tender rule, terms of a K for sale of goods are enforced exactly, every K term
treated as an express condition
 Rule: the seller is in breach if the goods fail in any respect to conform to the K
 If seller fails to make perfect tender, buyer has 3 courses of action available:
 Reject the Goods
o Must reject within reasonable time + notify seller
o Buyer may then sue for damages unless cure applies
o If buyer fails to reject in manner specified above, then it is deemed an acceptance of the
goods by the buyer

 Accept the Goods


o Occurs when buyer had reasonable time to inspect + signifies acceptance by:
 Stating that goods conform to the K;
 Taking the goods despite non-conformance;
 Failing to make effective rejection; OR
 Taking any action inconsistent with seller’s ownership of goods (like putting goods on
the showroom floor)
o Legal consequences of buyer’s acceptance:
 Buyer must pay K price
 Buyer may seek damages for any non-conformity if seller has been seasonably
notified
 Buyer can revoke acceptance if non-conformity substantially impairs value of the
goods and if (1) buyer accepted because unaware of non-conformity, OR (2) seller
assured cure but failed to cure

 Accept Part and Reject Part - Buyer can only do so in terms of commercial units of that
good (can’t accept half a loaf of bread only)

Notes:

37
MBE CONTRACTS

 Work-Arounds for Breaching Sellers


 Right to Cure (before K deadline): If seller makes non-conforming tender but time for
performance has not passed, then the seller may substitute conforming goods
o Two requirements:
 Seller must give buyer reasonable notice of intent to cure; and
 Seller must make conforming delivery by K deadline

 Right to Cure When There Were Reasonable Grounds to Believe Delivery Was
Acceptable (after K deadline):
o Belief must be based on buyer’s express assurances, trade usage, course of dealing, or
course of performance
o If such grounds for belief exist, there are two requirements for seller’s cure:
 Seller must give buyer reasonable notice of intent to cure; and
 Seller must make conforming delivery within reasonable time

 Imperfect Tenders in Installment Contracts


 Under UCC, installment contracts are contracts that contemplate delivery of goods in
separate lots to be separately accepted by the buyer
 Perfect tender rule DOES NOT apply to installment Ks. Buyer must generally give the seller
an opportunity to cure any defects in the installment deliveries, unless the seller
“substantially impairs” the value of the entire K.

Notes:

38
MBE CONTRACTS
DEFENSES

1) Incapacity
 Infancy/Minors
o Modern Rule: Minors (under 18) may enter into a K, but it is voidable at minor’s option
 Majority of jurisdictions - neither of the following affect minor’s avoidance rights:
 Marriage or emancipation
 Misrepresenting the minor’s age
o Power of Avoidance (or Disaffirmation): If minor exercises right to disaffirm K:
 Minor is obligated to return any goods received but not liable for any damages or reasonable
value for use of any goods or services
o Ratification: Once minors turn 18, they may expressly or impliedly ratify Ks entered during minority
and bind themselves to obligations they might otherwise have disaffirmed
o Necessaries Exception: A minor’s K for necessaries (food, clothing, shelter, medical care) is
voidable, but merchant has quasi K right to recover reasonable value of goods or services

 Mental Incompetence
o Under modern rules, a person lacks capacity to contract if mentally incompetent (i.e., to reasonably
understand the transaction) at time of contracting.
o Power of Avoidance (or Disaffirmation): If mentally incompetent person exercises right to
disaffirm K:
 (unlike minors) the mentally incompetent person is obligated to not only return any goods
received but IS also liable for any damages or reasonable value for use of any goods or services
(unless other party was aware of incompetence then mentally incompetent parties are treated
just like minors)
o Ratification: a party who was mentally incompetent at the time of contracting may expressly or
impliedly ratify the K if he becomes competent at a later time.
o Necessaries Exception:
 The mentally incompetent party can disaffirm the K but the provider of the necessaries can
recover in quasi-contract for the reasonable value of the goods/services
 Additional necessary – legal representation in connection with incompetency proceedings.

 Incapacity defenses are available under common law and the UCC.

Notes:

39
MBE CONTRACTS
2) Misrepresentation
 Fraudulent Misrepresentation (4 elements)
o Element #1: A Misrepresentation
 Defendant must have made an assertion inconsistent with existing facts
 What would be sufficient to prove this element?
 Oral or written misrepresentations
 Fraudulent conduct such as concealment
 A half-truth
EXAMPLE: A seller of real estate undertakes efforts to hide termite damage.
 Misrepresentations are not:
 Broken promises, but rather only misstatements of existing fact
 Opinions or guesses
o Element #2: State of Mind (need both scienter and intent)
 Scienter is satisfied if defendant made assertion either knowing it to be false OR knowing he
had no idea whether it was true or false; AND
EXAMPLE: Auto dealer says, “this baby can go from zero to 70 in six seconds flat.” If he either
knows that the car can’t accelerate that quickly, or has no basis for knowing whether the car can
accelerate that quickly, scienter is present.
 Intent to Mislead is satisfied if defendant made assertion for purpose of misleading the
aggrieved party OR knowing there was a substantial likelihood to mislead
o Element #3: Materiality of Misrepresentation
 Objective Materiality: where such an assertion is likely to induce a reasonable person to enter
into a contract
EXAMPLE: An assertion that a used car has been carefully inspected by an independent auto
mechanic would appeal to a reasonable person, and thus satisfy the objective materiality test.
 Subjective Materiality: if the party making the assertion had reason to know that it was likely to
induce the particular aggrieved party into entering the K
EXAMPLE: An assertion that the car was once owned by the actor Jon Voight would be
material if made to an individual (e.g., George Costanza) whom the seller knew to be a Voight
enthusiast.

Notes:

40
MBE CONTRACTS
o Element #4: Reasonable Reliance on the Misrepresentation
 What counts as unreasonable reliance?
EXAMPLE: Reliance would be unreasonable if the aggrieved party has independent knowledge
or reason to know that the statement in question is false.
EXAMPLE: Reliance would be unreasonable if the aggrieved party has reason to believe that
the statement in question was made by a person who is unreliable.
EXAMPLE: Reliance would be unreasonable if no reasonable person would have believed the
assertion.
EXAMPLE: Reliance would be unreasonable if the aggrieved party could have easily
ascertained the truth by cursory inspection of the goods.

 Non-Fraudulent Misrepresentation – 2 types, Negligent and Innocent misrepresentation


o Element #1: A misrepresentation
o Element #2: Materiality of the misrepresentation
o Element #3: Reasonable reliance on the misrepresentation
o But #4 is different: we don’t need scienter/intent to mislead, but rather:
 For Negligent misrepresentation: the defendant would have known the assertion was false had
he exercised reasonable care
 For Innocent misrepresentation: the defendant made an assertion not in accord with existing
facts

 Fraudulent Nondisclosure: The fraud consists of defendant’s silence when duty to disclose
o Element #1: the nondisclosure was material to the K;
o Element #2: reasonable reliance on nondisclosure;
o Element #3: a duty of disclosure and failure to fulfill it
 Although there is generally no duty of disclosure to trading partners, if a party is aware of
material facts that are unlikely to be discovered by the other party in the exercise of ordinary
care and diligence, then there will be a duty to disclose that information in these circumstances:
 Parties enjoy relationship of trust and confidence (e.g., familial relationships or the
relationship between a professional and client)
 Party has made an assertion that was true at the time but has been rendered untrue by
intervening events
 If obligation of good faith would require that the party disclose the information (e.g., real
estate transaction where one party knows of a termite infestation)
 Note: Defenses are equally applicable under common law and the UCC.

Notes:

41
MBE CONTRACTS
3) Duress Defense (3 elements)
 Element #1: A threat:
 Element #2: that is wrongful in nature
o 3 types or circumstances:
 What is threatened is a crime or tort
 What is threatened is a criminal prosecution or bad faith civil process
 What is threatened is a bad faith breach of K (aka economic duress)
 NOTE: economic duress is the most common circumstance tested
EXAMPLE: Sailors on a fishing voyage cease their work midway through the voyage and refuse to
resume their duties unless the captain agrees to give them a raise.
 Element #3: No reasonable choice but to succumb to the threat:
o This is seldom an issue when what is threatened is a crime or a tort, a criminal prosecution, or a bad
faith civil suit, for risking the eventuality of any of those is scarcely a “reasonable” choice.
oHard cases often arise in cases involving “economic duress” (i.e., threatened breach of contract) and
commonly arise in these three situations:
 Situation #1: no adequate and reasonably priced substitutes for the services/goods that are
threatened to be withheld
EXAMPLE: The ship’s captain is not in a position to find replacement sailors when already at
sea, so no adequate substitutes would be available.
 Situation #2: the threatened breach would cause aggrieved party to break his own K’s (i.e. a
threatened breach of a subcontractor that may force the general contractor to breach his own
Ks)
EXAMPLE: A purchaser of upsidaisium bearings used in the manufacture of turbo-jet engines
would, on account of the breach, be forced to breach its contract with an aircraft manufacturer.
 Situation #3: When the alternative of acquiescing to the threat and then suing for damages is
inadequate to address the harm caused
EXAMPLE: Where the threatened breach would cause the aggrieved party to renege on its own
commitments and thus harm its reputation and opportunities for future business, damages
would be inadequate as a remedy.
 The common law defense of duress can be used in cases arising under the UCC as a supplementary
provision.

Notes:

42
MBE CONTRACTS
4) Undue Influence (2 elements)
 Element #1: Unfair persuasion used (multi-factor test for this element)
o Discussion of the transaction at an unusual or inappropriate time
o Consummation of the transaction at an unusual place
o Insistent demands that the transaction or business be finished immediately
o Extreme emphasis on the untoward consequences of delaying the transaction
o The use of multiple persuaders against the target of persuasion
o Absence of third-party advisors to the target of persuasion
o Statements that there is no time to consult financial advisors or attorneys
 Element #2: Other party was vulnerable to such persuasion
o A vulnerable party can be established if one of the following exists:
 Circumstance #1: mental infirmity is due to age or illness
 Circumstance #2: vulnerability is due to recent trauma or event
 Circumstance #3: there is a relationship of trust or confidence
 Like the other defenses, the claim of common law undue influence is available in cases arising under
the UCC as a supplementary provision.
5) Unconscionability (2 elements)
 Element #1: Procedural Unconscionability: Bargaining process created an absence of meaningful
choice for aggrieved party
 Element #2: Substantive Unconscionability: K terms are unreasonably favorable to one party to
the K (ex: excessive price, extreme consequences for breach, or grossly unfair provisions)
EXAMPLE: Where a pay-over-time plan requires the consumer to pay a total sum that is many
times the value of the purchased goods; or where a bank charges an overdraft fee that is many
times the bank’s actual processing costs.
EXAMPLE: An add-on clause, pursuant to which a merchant is entitled to repossess multiple
household items, including furniture, bedding, and a stereo, when a consumer missed a payment,
despite the fact that the consumer had already paid nearly 80% of the monies owed for the various
purchases.
 Legal Consequences – Upon a finding of unconscionability, court may:
o Refuse to enforce K;
o Excise the offending clause and enforce remainder;
o Limit application of offending clause to avoid unconscionable result

Notes:

43
MBE CONTRACTS

6) Public Policy (can be raised in 3 contexts)


 Context #1: Subject of K itself is specifically prohibited by law
EXAMPLE: In most jurisdictions, a contract for prostitution, gambling, or bribery is illegal under the law
and accordingly unenforceable in court.
 Context #2: K formed for purpose of committing a crime or tort
EXAMPLE: A contract between an employer and a hired assassin would be a contract for the
commission of a crime.
 Context #3: K performance would violate certain values/freedoms designated by the state
EXAMPLE: A contract that prohibits one party from marrying for an extended period of time would
violate the public policy of promoting free and consent-based marriages.

Notes:

44
MBE CONTRACTS
REMEDIES

1) Remedies for Breach of Contract – Generally


 The general rule in American contract law is that a remedy for a breach of contract is a payment of
expectation damages. *That should be the starting point of every analysis – this is the default award.
 Monetary Damages at Common Law
o Expectation Damages (court’s default): Aggrieved party will be entitled to amount that will restore
him to the position he would have been in had the K been fully performed
EXAMPLE: A patient with a hand injury contracts with his doctor for hand surgery. The doctor
promises him a “100% perfect” hand, but instead the surgery makes the hand worse. The
expectation interest of the patient is measured by the difference between the value of the promised
“100% perfect” hand and the value of the hand worsened by surgery.
o Calculating the Monetary Award
Formula:

Loss of value of the breaching party’s performance


+ any incidental and consequential costs generated by the breach
– any payments received from the breaching party
– any costs saved as a result of the breach

= Expectation Damages of the Aggrieved Party

EXAMPLE: General contractor repudiates its contract with Subcontractor when Subcontractor is
halfway finished with its work under the subcontract. Subcontractor is entitled to seek from the
breaching General contractor an amount equal to the contract price or the unrealized value of
General’s promised performance, plus costs associated with storing, insuring, and/or returning
materials and equipment secured by Subcontractor in the course of contractual performance (i.e.,
incidental costs generated by the breach) or the incidental costs generated by the breach; minus any
progress payments already made by General contractor; minus money Subcontractor may have
saved on salaries and equipment rental by not having to complete performance.

Notes:

45
MBE CONTRACTS
 Limitations on Right to Recover Expectation Damages
o Aggrieved party may not be able to recover the full amount in the following situations:
 Situation #1: expectation damages can’t be calculated with reasonable certainty
 Mathematical certainty is not required, but the court can’t pull the numbers from thin air
EXAMPLE: A business with no profit history
 Situation #2: damages are unforeseeable

HYPOTHETICAL: Supplier agrees to deliver a quantity of window glass to Store for $1,000 on a certain date.
Store needs the glass to fill its depleted inventory. Supplier fails to deliver as promised because the market
price of the quantity of glass has risen to $1,200 and Supplier prefers to sell to a third party at the higher price.
Unbeknownst to Supplier, Store had, in the meantime, entered a contract to sell the entire quantity of glass for
$1,500 to a business services firm for use in the many properties it manages, and Store is unable to deliver
because of Supplier’s breach. To what general damages is Store entitled? Suppose Store’s buyer explained at
the time of contracting that it needed timely delivery of the glass in question because it had already committed
to selling the quantity in question to a third party. How would this change the analysis? Suppose the buyer of
the glass is not a store but a glazing contractor that needs the glass in question to install windows in a new
commercial building, and it will be required to pay liquidated damages to the general contractor if it fails to
perform in a timely fashion. How would this change the analysis?

Notes:

46
MBE CONTRACTS
 Situation #3: where damages can be mitigated
EXAMPLE: The liability of an employer is reduced if an employee dismissed in breach of
contract takes an extended vacation rather than seeking other work. But the aggrieved
employee need only make reasonable efforts to secure a position that is reasonably equivalent
to the job lost, and accordingly she need not accept substitute work when it: is in a different
field; offers significantly lower pay or less desirable terms and conditions of employment than
those of the lost job; would entail more burdensome responsibilities than those of the lost job; or
would damage the aggrieved party’s career prospects. (e.g., an A-list movie star would not be
required to mitigate damages for a film producer’s breach of contract by taking a lesser role in a
B-grade film).

 Reliance Damages: designed to restore the aggrieved party to the position he was in prior to the K.
o Aggrieved party’s reliance interest is measured by:
 Any expenditures made in preparation for performance or in actually performing
 Less: any loss which the breaching party can prove the aggrieved party would have suffered
even if the K had been fully performed
EXAMPLE: A agrees to buy B’s car. On the faith of the promise, A pays B a $2,000 down
payment for the car and enters a 12 month lease agreement with C for a parking space for
$2,400. The reliance interest of A would be $4,400, the amount it would take to compensate A
for what he’s lost on the faith of B’s promise.
o Reliance damages may be available where expectation damages are not available, such as when
they are too uncertain or speculative.
EXAMPLE: Inventor signs a contract with Railroad Co. for shipment of a new stove Inventor plans
to display at a manufacturers’ convention, and she pays fees to the convention hotel for a room and
for display space, as well as an exhibition fee to the convention sponsor. If Railroad Co. breaches
the contract by failing to deliver the stove until after the convention, Inventor will be unable to
recover damages for opportunities she may have lost due to her inability to showcase the stove, as
calculating the probability and value of those opportunities would be too speculative. However,
Inventor would be entitled to reliance damages (i.e., a return of the Railroad Co.’s fee, plus
compensation for the amounts she paid to the hotel and convention sponsor).

Notes:

47
MBE CONTRACTS
 Restitutionary Damages: the value of benefits conferred upon other party in the transaction
EXAMPLE: Where the buyer of a car makes a $2,000 down payment to the seller and a $2,400
payment to the lessor of a parking space, the buyer’s restitutionary interest is limited to a recovery of
the $2,000 down payment because the parking space rental payment was made to a third party.
Although both of these “out of pocket” expenses are recoverable as reliance damages, only the down
payment is available as restitution.

o Calculation - Court’s discretion and as justice requires by either:


 Reasonable value or cost of benefits conferred; OR
 Extent to which the other party’s property has increased in value because of the performance
rendered
EXAMPLE: Contractor built a Turkish bathhouse on Homeowner’s property. The market value
of the services is the cost to Homeowner of hiring another contractor to do the work, which
equals $30,000. The value added to Homeowner’s property is the net increase in the home’s
value from the addition of the bathhouse, which equals $20,000. A court calculating Contractor’s
restitutionary interest could award him either amount, considering what relevant precedents
permit and what justice requires.

 Aggrieved party is likely to elect to recover restitutionary damages when it would exceed the
amount recoverable based on his expectation interest, and that is most likely to arise in the
context of a “losing” K.
EXAMPLE: General contractor breaches its contract with Subcontractor when Subcontractor is
halfway finished with its work under the subcontract. Subcontractor is entitled to seek from the
breaching General either expectation damages or the contract price less any amounts already
paid and also less any savings due to the shortened performance. Alternately, the party could
seek restitutionary damages or the market value of the services already rendered at the time of
breach. Even if Subcontractor had a “losing contract,” it is nevertheless entitled to recover its
restitutionary interest.

 Important Limitation: Available if aggrieved party has partially performed but NOT if the party
has fully performed
EXAMPLE: Subcontractor completes contractual performance and General Contractor refuses
to pay. Subcontractor has no right to seek restitutionary damages and is entitled to recovery of
the contract price only.

Notes:

48
MBE CONTRACTS

 Liquidated Damages Provisions (LDP): Designed to provide damages of their own choosing in the
event of breach
o Such a provision is:
 Enforceable if court finds it to be valid liquidated damages clause designed to compensate for
breach
 Unenforceable if court finds it constitutes a penalty designed to punish a breach
o LDP Test: two primary prongs, either can uphold the clause
 Prong #1: was the clause reasonable at the time of contracting in relation to anticipated harm
 Key question: whether there was an anticipated harm that would be difficult to prove?
EXAMPLE: Repudiation of a commercial lease by an “anchor” tenant in a shopping mall is likely
to cause reductions in pedestrian traffic to other stores as well as harm to the mall’s reputation
and attractiveness as a shopping “destination.” Because the extent and monetary value of those
harms would be difficult to establish in court, a liquidated damages clause designed to award
the mall owner a sum that represents an inexact but reasonable forecast of those harms would
be appropriate.

 Prong #2: was the clause reasonable in relation to the harm and losses that actually occurred
due to the breach?
EXAMPLE: If the shopping mall tenant repudiates the contract shortly after entering it, and the
mall owner is able to secure a substitute “anchor” with minimal delay, a clause guaranteeing the
owner significant liquidated damages would be unreasonable in relation to the actual harms
caused by the breach.
o If courts find it is in fact a penalty  strike from K, damages in accordance with default rules

Notes:

49
MBE CONTRACTS
 Monetary Damages under UCC
o Seller’s Remedies: Right to recover depends on whether the goods were delivered and accepted
by the buyer
 Goods delivered to and accepted by Buyer  Remedy is K price
 Not delivered and accepted (because buyer wrongfully rejected or repudiated), then the seller
can recover damages and the measure depends on whether seller has resold goods to a third
party.
 If seller has re-sold  damages are difference between K price and resale price
 If seller has not re-sold  difference between K price and market price
 Lost Volume Sellers = Supply of goods exceeds the demand
o Recover the profit they would have made on the lost sale
o Buyer’s Remedies: Depends on whether buyer has covered (bought replacement goods)
 If buyer covers (in good faith)  difference between K price and cover price
 If buyer does not cover  difference between K price and market price

 Difference in Value Damages for Accepted Goods - available if buyer receives and accepts
nonconforming goods from the seller
 Buyer is entitled to recover the difference between value of the goods contracted for AND
value of the goods received
o Incidental Damages:
 Buyer or seller can recover incidental damages, which are the expenses incurred in either seller
dealing with the goods after buyer’s breach (e.g., storing them) or buyer arranging for cover
(e.g., transportation costs)
o Consequential Damages:
 Under the UCC, the buyer can additionally recover consequential damages that were
reasonably foreseeable to the seller at the time the contract was entered into (e.g., lost profits
because of delay in obtaining goods). Article 2 does not explicitly give seller right to
consequential damages.

Notes:

50
MBE CONTRACTS
 Equitable Remedies
o First type: Specific Performance
 Extraordinary remedy by which breaching party is ordered to perform
 Only available when monetary award is considered inadequate
 Where Specific Performance is Generally Available:
 Money damages are generally presumed inadequate when party is purchasing:
o Unique objects (works of art and precious heirlooms)
o Real Property
 Where Specific Performance is not Available: Contracts for:
 Personal services
 Long-term relationships
 Specific Performance under the UCC
 The UCC liberalizes the rules governing the availability of specific performance
o “Uniqueness” requirement  Need only adequately search and be unable to find
reasonable substitutes, and so is unable to “cover” the breach by seller
o “Capable of immediate performance” requirement  specific performance allowed in
output and requirements Ks (classic long-term relationships that require cooperation of
the parties)

Notes:

51
MBE CONTRACTS
o Second type: Negative Injunctions
 Orders prohibiting breaching party from doing something
 Most common and important area is the employment setting
 Availability turns on whether former employer is seeking mid-term or post-employment relief
 Mid-Term Relief = When employee under K for a specific period of time and breaches by
departing before the end of that period, a negative injunction will be available, even absent a
K prohibition, to prevent competing if employee’s services are unique or extraordinary (pro
athletes and entertainers)
EXAMPLE: In the famous case of Lumley v. Wagner, an opera singer under contract to sing
at Her Majesty’s Theatre for a three-month period was persuaded to depart mid-contract to
begin a concert series at a competing venue. Because her services were unique and
extraordinary, the court granted a negative injunction barring her from performing at any
competing venue for the duration of the contract term.

 Post-Employment Relief (Covenants not to Compete) = K provision that prohibits post-


employment competition.
o Validity depends on 3 factors/considerations:
 Consideration #1: is there a significant business justification for enforcing post-
employment restraints?
EXAMPLE: An employee with access to trade secrets and entrepreneurial
knowledge.
 Consideration #2: is the scope of the non-compete clause reasonable in
duration and geographical reach?
 Consideration #3: is there an express provision? (court won’t imply one)

Notes:

52
MBE CONTRACTS
2) Other Possible Remedies
 Promissory Estoppel: the type of interest a party may recover under promissory estoppel depends on
the jurisdiction, as some courts award:
o Expectation damages;
o Reliance damages; or
o Choose on a case-by-case basis and tailor the remedy to the injustice at issue

 Restitution and Unjust Enrichment


o Restitution: Recall this is an alternative to expectation damages that a party can recover for breach
of K
 However, in the following contexts restitution may be available even when the other party has not
breached (i.e., restitution is also a separate cause of action)
 Benefits Conferred under a Failed Contract: when a party bestows benefits on his trading
partner in connection with what turns out to be a “failed” contract (e.g., incapacity, fraud,
duress) the party bestowing the benefits may recover their value via restitution

 Benefits Conferred by a Breaching Party: a party that breaches a K may recover the
benefits it conferred on the non-breaching party, so long as there is an offset for any
damages caused by the breach
EXAMPLE: A works for B under a 12-month contract and, without advance notice, breaches
the contract after 11 months in order to other accept other employment. A is entitled to
restitution of the reasonable value of the services rendered, subject to offset for B’s
damages—that is, the cost of hiring a replacement for A at the last minute.

Notes:

53
MBE CONTRACTS

 General Unjust Enrichment (No Express Contract): a party that confers benefits on
another may recover their value where it is unjust for the recipient to retain the benefits
without paying, even absent any express or implied-in-fact K. Two recurring situations
where recovery has been held appropriate:
o Medical services provided by a medical professional; and
o Benefits conferred by mistake to one who availed himself of the benefits at issue
EXAMPLE: After a hurricane, Owner hires Contractor to perform repairs on his storm-
damaged home. Because of confusion caused by damage to street signs and
mailboxes, Contractor performs the repairs on the wrong house. Contractor is entitled to
restitution from the benefiting homeowner if the latter was aware of Contractor’s
mistaken efforts and remained silent.
o A person who bestows benefits without request by the benefitting party is considered an
“officious intermeddler,” not entitled to any recovery
 Exception: doctors and other health care professionals who provide emergency
health care to a patient unable to consent (e.g., because too ill or unconscious)

 Agreed-To Remedies:
o Parties may contract out of the legal and equitable remedies available under the law by specifying
agreed-to remedies in the K
o Two typical forms:
 Liquidated damages provisions; and
 Provisions limiting or excluding damages
o Provisions Limiting or Excluding Damages:
 Provisions that limit or alter the measure of damages available
EXAMPLE: An exclusion of consequential damages.
 Exclusive remedies, such as limits to repair or replacement of defective goods
 Such provisions are generally enforceable unless they are unconscionable or they fail of their
essential purpose
 Limitation of consequential damages for personal injury in the case of consumer goods is prima
facie unconscionable

Notes:

54
MBE CONTRACTS
THIRD-PARTY BENEFICIARIES

ILLUSTRATION #1:
The ----------------------> payment The
LAWYER painting <------------------ ARTIST

LAW SCHOOL LANDLORD


(donee beneficiary) (creditor beneficiary)
1) In General
 In some contracts, one of the parties promises a performance that will benefit a third party (a third-party
beneficiary)
 Critical issue: the circumstances under which the third-party beneficiary can sue to enforce the K.

2) Classification of the Third-Party Beneficiaries and Enforcement Rights:


 The right of an aggrieved third-party beneficiary to bring an action against a breaching promisor or
promisee depends on the classification of the beneficiary
 Intended Beneficiary: one whom the contracting parties intended to benefit (often named in the
contract).
o Creditor Beneficiary: Promisee seeks a performance from promisor to satisfy an obligation owed
to a third party
 The landlord in our diagram is a creditor beneficiary because artist’s K with lawyer is designed
to satisfy the artist’s debt to the landlord – i.e., lawyer promised artist he would pay artist’s debt
owed to LL)
o Donee Beneficiary: Promisee seeks performance from promisor in order to make a gift of that
performance to a third party
 The law school in our diagram is a donee beneficiary because lawyer’s K with artist is designed
to provide a gift of artist’s work from lawyer to law school

o Standing to Sue [Yes]. Intended beneficiaries can sue the promisors who directly promised to
provide the benefit, but can’t sue the promisees based on the K, although they can still sue on any
prior obligation (if any).
EXAMPLE: Law School can sue artist (the “promisor” who promised to render painting for school).
But Law School cannot sue lawyer (the “promisee” who obtained artist’s promise) if the painting
doesn’t happen (no prior obligation between lawyer and law school).
EXAMPLE: Landlord can sue lawyer (the “promisor” who promised to pay money to Landlord). But
Landlord cannot sue artist (the “promisee” who obtained lawyer’s promise) based on the lawyer-
artist contract, but can of course still sue based on back rent still due (prior obligation).

Notes:

55
MBE CONTRACTS

 Incidental beneficiary: third parties who will benefit from a promisor’s performance as a practical
matter, but are not intended beneficiaries.
EXAMPLE: The residents who live in the neighborhood where Lawyer’s school is located will be
benefited by the mural, but none of them can sue if Artist does not deliver because they are incidental
beneficiaries. Only the school has a claim.
o Standing to Sue [No]. Incidental beneficiaries cannot sue anyone to enforce the K.

3) Vesting of Third-Party Rights to Sue


 Parties to a contract are free to modify or rescind by mutual consent, and they may modify or rescind a
third party beneficiary provision without consent unless beneficiary’s rights under the K have vested.
 With intended beneficiaries, vesting occurs when:
o Situation #1: beneficiary brings suit on the matter

o Situation #2: beneficiary changes position in justifiable reliance on the K;


EXAMPLE: Landlord has had it with Artist and is ready to start eviction proceedings. The landlord
then learns that Artist has arranged that Lawyer’s payment for the mural be paid the landlord . In
light of this, the landlord decides not to continue with the conviction proceedings. At this point,
because the landlord/beneficiary has changed his position in justifiable reliance, the
landlord/beneficiary’s rights have vested.
o Situation #3: beneficiary manifests assent to the K at the request of promisor or promisee;
EXAMPLE: Patron (the lawyer) speaks to representatives of her alma mater, informing them about
the mural that she has arranged to be painted for the school. The representatives gladly assent to
the painting. Once the school says it will accept the mural, it is too late to modify the contract with
the artist.
o Situation #4: rights of beneficiary have vested under express K terms
EXAMPLE: Under the terms of her life insurance policy with Hartford Insurance, A’s designations of
beneficiaries become irrevocable upon A’s demise or adjudication of incompetence. The rights of C,
a designated beneficiary, against Hartford Insurance will vest upon the occurrence of either event.

Notes:

56
MBE CONTRACTS
 Defenses Available to Promisor: any valid defenses the promisor has against the promisee are also
effective against the third-party beneficiary
EXAMPLE: If Lawyer and Artist enter into a contract in which Lawyer agrees to pay artist for the
painting of a wall mural, and Artist in fact does not paint that mural, there is a failure of consideration
and Lawyer’s obligation to pay is discharged. Moreover, Lawyer can raise that as a defense against
Artist’s landlord, if that landlord sues as a third-party beneficiary of the contract.

o Promisor may not assert defenses based on separate transactions with the promisee.
EXAMPLE: Before Lawyer entered into a contract with Artist for the painting of the mural, Lawyer
had loaned Artist money. Artist has not repaid the loan as promised. In an action between Artist and
Lawyer, Lawyer has a right of setoff. But if Artist’s landlord sued as a third-party beneficiary to the
contract for the mural, Lawyer could not raise the setoff as a defense.

Notes:

57
MBE CONTRACTS
ASSIGNMENT OF RIGHTS AND DELEGATION OF DUTIES

ILLUSTRATION #2:

The ---------------------> mortgage payments The


DEBTOR loan <------------------ BANK
(obligor) (assignor)

The
MORTGAGE CO.
(assignee)

1) Assignment of Rights
 Assignment = A transfer of a right to receive a contract performance
 To be effective, owner of the right must manifest an intention to make a present transfer of an existing
right

 Rights to Be Assigned: all rights generally assignable except when:


o Exception #1: when assignment would materially change the duties of other party;
EXAMPLE: A and B are parties to a requirements contract under the terms of which A is obligated
to supply B with B’s monthly requirements for widgets. B’s rights under the contract are not
assignable to C if C’s monthly requirements would greatly exceed B’s.
o Exception #2: when the obligor has personal interest in rendering performance to the obligee and
not a third party;
EXAMPLE: A student attempts to assign his right to a private tutor who is picky about his students.
Because the tutor has a personal interest in who he tutors, the contract is not assignable.
o Exception #3: when it would violate applicable law or public policy;
EXAMPLE: A state statute prohibiting assignment of wages.
o Exception #4: when assignment is prohibited by the K
 However, most courts will treat this as a breach of K by assignor, but not a basis for nullifying
assignee’s rights
 EXCEPTION: If the contract provision provides that any assignments of this contract “ARE VOID” or
similar language, than any assignment is completely negated.

Notes:

58
MBE CONTRACTS

59
MBE CONTRACTS

 Assignment for Value versus Gratuitous Assignment


o Assignment for value: valid against obligor and cannot be revoked by assignor
o A gratuitous assignment has the following legal effects:
 Between assignee and obligor  Valid against obligor, who cannot claim lack of consideration
as a defense
 Between assignor and assignee  Under law of gifts, an executory gift is revocable but an
executed gift is not. A gift is executed when there is intention coupled with actual or symbolic
delivery
EXAMPLE: A recent graduate of law school has paid for a bar course but decides instead to
join the clergy. As a gesture of love, he transfers the right to the bar course to a friend not for
value but as a gift.

 Rights and Obligations of the Parties after Assignment


o Rule:
 An assignee gets whatever rights to the K his assignor had; AND
EXAMPLE: If assignor paid for the full course, that is what the assignee is entitled to.

 An assignee takes subject to whatever defenses the obligor could have raised against the
assignor
EXAMPLE: If assignor has not paid for the full course, the bar course can raise that as a
defense against the assignee.

o Payment to Assignor: obligor’s payment to assignor is a defense unless the obligor has been notified
that the payments are now owed to the assignee

Notes:

60
MBE CONTRACTS

ILLUSTRATION #3:

2) Delegation of Duties
 Delegation: occurs when a third party agrees to satisfy a performance obligation owed by one of the
parties to a contract
 Rights of the Obligee against the Delegator
o A delegation of duties does NOT operate as a “transfer” of duties from delegator to delegatee
o Absent a novation (agreed substitution of one obligor for another), a delegation does not
relieve the delegator from his obligations under the K (obligee can still sue delegator)
 Liability of the Delegatee
o To the delegator (the Professor in our example)
 If delegation was for consideration, delegator has a breach of K action against delegatee who
doesn’t perform
o To the obligee (the bar course in our example)
 If delegation was for consideration, obligee can bring action against delegatee as an intended
third-party beneficiary (a creditor beneficiary) of the delegation
 Delegable Duties: generally, all contractual duties are delegable
o Exception #1: personal services
EXAMPLE: Nanny is under contract to provide child care services to B’s family. Her performance
obligation cannot be delegated to a third party.
o Exception #2: when the K prohibits delegation

Notes:

61
MBE CONTRACTS

RECAP QUESTIONS:
Please take a few minutes and answer the following questions. Try to answer these questions without looking
at your Bar Notes. If you must go back to your notes, try rewriting the answer in your own words. This
exercise will help improve your memory retention of these concepts.

1. What are the three factors used to determine whether the predominant purpose of a contract is for the sale of
goods or not?

2. What are the two requirements of a valid offer?

3. Describe the four ways to terminate the power of acceptance of an offer.

4. What are the two requirements of acceptance?

5. Explain the Mailbox Rule.

6. What is the UCC solution to the “Battle of the Forms”?

7. What are the four requirements that must be met for promissory estoppel to be available?

62
MBE CONTRACTS
8. What are the six categories of agreements covered by the Statute of Frauds?

9. What are the five ways one can satisfy the UCC Statute of Frauds?

10. Under the UCC, where there are pertinent terms missing, what are the default rules used to fill a missing price
term, missing time term (e.g., time/date of delivery or time/date for action taken under the contract), and a
missing place of delivery term?

11. What are the three purposes for which parol evidence is introduced?

12. When will the parol evidence rule not apply?

13. When does the risk of loss transfer in carrier cases? In non-carrier cases?

14. What is the preexisting duty rule?

15. What is the perfect tender rule? What three options are available to the buyer if this rule is violated?

16. With regard to the defense of incapacity, what is the rule regarding infants and minors?

63
MBE CONTRACTS

17. What are the elements of duress?

18. What are expectation damages?

19. Describe three situations when the right to recover full expectation damages may not be available.

20. When is specific performance NOT available?

21. Who are the parties in a Third Party Beneficiary scenario?

22. In what instances are contracts rights not assignable?

23. After a delegation of duties, what obligations , if any, remain with the delegator?

64

You might also like